Sunday 16 February 2014

abstract algebra - Is this a theorem regarding the solutions of polynomials?



I wanted to refer to this, but I can't remember if this a theorem, named or otherwise, and if it is, how to properly state it. The idea is if we have a solution in radicals to a polynomial with integer coefficients, then where we have a $n^{th}$ root expression within the solution we can multiply that expression by any $n^{th}$ root of unity and this will also be a solution.



i.e. if $p(2+\sqrt{3+\sqrt7})=0$ then




$p(2+\sqrt{3-\sqrt7})=p(2+\sqrt{3+\sqrt7})=p(2-\sqrt{3-\sqrt7})=0$



if $p(2+\sqrt{3+\sqrt[3]7})=0$ then



$p(2+\sqrt{3+\sqrt[3]7})=p(2-\sqrt{3+\sqrt[3]7})=p(2+\sqrt{3+\sigma_{1}\sqrt[3]7})=p(2-\sqrt{3+\sigma_{1}\sqrt[3]7})=p(2+\sqrt{3+\sigma_{2}\sqrt[3]7})=p(2-\sqrt{3+\sigma_{2}\sqrt[3]7})=0$



Where $\sigma_{1} = e^{\frac{2}{3} \pi i}$, $\sigma_{2} = e^{\frac{4}{3} \pi i}$



I think this relates to Field Theory where these other solutions are the 'conjugates' of polynomial used for extension of a field. But that's pretty fuzzy to me nowadays. I may just have guessed at this when I studied it a long time ago.




Also - this set of solutions is the set of solutions for an integer polynomial which will be a factor of $p$?



Also - possibly we not even had to suppose the original expression was a solution, this tells us how to construct the required polynomial?


Answer



There are some limitations to the combinations of the roots of unity that can be used. Nested simplifiable square roots provide some. Also with cubics. Consider the polynomial
$$
p(x)=x^3-6x-6.
$$
Its zeros are

$$
x_1=\root3\of2+\root3\of4,\quad x_2=\sigma_1\root3\of2+\sigma_2\root3\of4,\quad x_3=\sigma_2\root3\of2+\sigma_1\root3\of4.
$$
But only these combinations of third roots of unity work. See a derivation of Cardano's formula for an explanation (or review Galois theory).


No comments:

Post a Comment

real analysis - How to find $lim_{hrightarrow 0}frac{sin(ha)}{h}$

How to find $\lim_{h\rightarrow 0}\frac{\sin(ha)}{h}$ without lhopital rule? I know when I use lhopital I easy get $$ \lim_{h\rightarrow 0}...